Difference between revisions of "1989 AIME Problems/Problem 1"
m (fmt, box) |
m (c) |
||
Line 3: | Line 3: | ||
== Solution == | == Solution == | ||
− | Let's call our four [[consecutive]] integers <math>(n-1), n, (n+1), (n+2)</math>. Notice that <math> | + | Let's call our four [[consecutive]] integers <math>(n-1), n, (n+1), (n+2)</math>. Notice that <math>(n-1)(n)(n+1)(n+2)+1=(n^2+n)^2-2(n^2+n)+1 \Rightarrow (n^2+n-1)^2</math>. Thus, <math>\sqrt{(31)(30)(29)(28)+1} = (29^2+29-1) = \boxed{869}</math>. |
== See also == | == See also == | ||
{{AIME box|year=1989|before=First Question|num-a=2}} | {{AIME box|year=1989|before=First Question|num-a=2}} | ||
+ | |||
+ | [[Category:Intermediate Algebra Problems]] |